Thứ Sáu, 4 tháng 12, 2015

MGMT 3304 - For Final exam 2015

Chapter 22


Performance and Breach of
Sales and Lease Contracts





N.B.:  TYPE indicates that a question is new, modified, or unchanged, as follows.

N   A question new to this edition of the Test Bank.
+   A question modified from the previous edition of the Test Bank,
=   A question included in the previous edition of the Test Bank.



true/false questions

1.   The standard of good faith is only read into contracts that provide ex­pressly for it.

     ANSWER:   F            PAGE:  425          TYPE:  +

2.   A seller’s tender of goods that does not conform to a contract is not a valid tender.

     ANSWER:   T            PAGE:  426          TYPE:  +

3.   Goods may be tendered in installments if the parties prefer it that way.

     answer:   T            PAGE:  426          TYPE:  +

4.   Under a destination contract, a seller must deliver the goods to a carrier, after which the risk of loss passes to the buyer.

     answer:   f            PAGE:  427          TYPE:  =

5.   A seller cannot exercise the right to cure once the time for performance under the contract has expired.

     answer:   t            PAGE:  427          TYPE:  N


6.   A substitution of one commercially reasonable carrier for another can be a valid tender of delivery if it is done within the time for performance.

     answer:   T            PAGE:  428          TYPE:  N

7.   In an installment contract, a buyer can reject any installment for any reason.

     ANSWER:   F            PAGE:  428          TYPE:  +

8.   If a nonconforming installment substantially impairs the value of a whole contract, the buyer can treat the entire contract as having been breached.

     answer:   T            PAGE:  428          TYPE:  N

9.   A delay in delivery of goods under a sales contract is a breach if perform­ance is commercially impracticable.

     answer:   F            PAGE:  429          TYPE:  N

10.  Unless the parties agree otherwise, the buyer must make payment at the time and place that the goods are received.

     answer:   T            PAGE:  431          TYPE:  N

11.  If goods fail to conform to a contract in any way, the buyer or lessee can­not make even a partial acceptance of the goods.

     ANSWER:   F            PAGE:  433          TYPE:  N

12.  A buyer’s breach of a contract will not usually give the seller the right to cancel the contract.

     answer:   f            PAGE:  435          TYPE:  N

13.  When a buyer breaches a contract while the seller is still in possession of the goods, the seller can resell the goods and hold the buyer liable for any loss.

     ANSWER:   T            PAGE:  435          TYPE:  =

14.  An unpaid seller can bring an action to recover the purchase price, on the buyer’s breach of a contract, only if the goods are first resold or oth­erwise disposed of.

     ANSWER:   F            PAGE:  436          TYPE:  =


15.  If a lessee wrongfully refuses to accept goods that conform to a contract, the lessor must tender substitute goods.

     answer:   F            PAGE:  436          TYPE:  N

16.  If a lessee wrongfully refuses to accept goods that conform to a contract, the seller can maintain an action to recover the damages sustained.

     answer:   T            PAGE:  436          TYPE:  N

17.  A buyer who obtains substitute goods to replace goods that a seller did not deliv­er can also recover damages from the seller.

     ANSWER:   T            PAGE:  439          TYPE:  +

18.  If a lessor’s tender of delivery fails to conform to a contract in any way, the lessee cannot accept the goods.

     ANSWER:   F            PAGE:  440          TYPE:  N

19.  A buyer who rightfully rejects nonconforming goods can resell the goods and keep the proceeds without accounting to the seller for any of the amount.

     ANSWER:   F            PAGE:  440          TYPE:  N

20.  A buyer who accepts nonconforming goods cannot revoke the acceptance.

     ANSWER:   F            PAGE:  441          TYPE:  N



multiple choice questions

1.   Superb Resources, Inc., sells unprocessed minerals to commercial proc­essors in Texas. With regard to the UCC’s good faith requirement, Superb can

a.   avoid it only by a conspicuous written disclaimer.
b.   avoid it only by oral disclaimer.
c.   avoid it with or without a disclaimer.
d.   not disclaim it.

     answer:   D            PAGE:  425          TYPE:  N


2.   International Gem Corporation agrees to sell Jewelry Outlets, Inc. (JOI), fifty new diamonds, but the contract does not specify a place of delivery. JOI is expected to pick up the goods. The place of delivery is

     a.   International’s place of business.
     b.   JOI’s place of business.
     c.   the Annual Gems and Jewels Convention.
     d.   the U.S. Postal Service office nearest to JOI’s place of business.

     answer:   A            PAGE:  426          TYPE:  N

3.   Custom Tableware, Inc., contracts for a sale of cutlery to Diners Cafe un­der a shipment contract. Custom must

a.   contract for transportation of the cutlery and tender to Diners documents to obtain its possession.
b.   neither contract for transportation of the cutlery nor tender to Diners documents to obtain its possession.
c.   only contract for transportation of the cutlery.
d.   only tender to Diners documents to obtain pos­session of the cutlery.

     ANSWER:   A            PAGE:  426          TYPE:  +

4.   Pep Paints agrees to sell to Quality Painters Grade A-1 latex outdoor paint to be delivered May 8. On May 7, Pep tenders Grade B-2 paint. Quality re­jects the Grade B-2 paint. Two days later, Pep tenders Grade C-3 paint with an offer of a price allowance. Pep has

a.   additional, unlimited time to cure.
b.   a reasonable, additional time to cure.
c.   one more day to cure.
d.   no more time to cure.

     answer:   D            PAGE:  427          TYPE:  N

5.   In Case 22.3, Banco International, Inc. v. Goody’s Family Clothing, the court held that anticipatory repudiation was indicated by

a.   Banco’s failure to start performance within a reasonable time to meet the contract deadlines.
b.   Banco’s misrepresentations concerning the status of its performance.
c.   Both a and b.  
d.   None of the above.

     ANSWER:   C            PAGE:  433          TYPE:  N


6.   On May 1, A-Plus Auto Sales agrees to sell a car to Bob. Five days later, Bob cancels the contract. A-Plus is enti­tled to

a.   force Bob to take the car.
b.   recover any resulting damages from Bob but not resell the car.
c.   resell the car and recover any resulting damages from Bob.
d.   resell the car but not recover any damages from Bob.

     answer:   C            PAGE:  435          TYPE:  N

Fact Pattern 22-1 (Questions 7–8 apply)
Acme, Inc., buys scrap metal from Beta Resources, Inc., to process and sell. Their contract provides for an annual review of the price. When the processed scrap’s market value decreases, the parties continue to ship and process the scrap while they review the price. During the negotia­tions, Acme does not pay Beta. Unable to agree on a price, Beta ends the deal, retrieves the scrap that was shipped and processed but not paid for, and sells it.

7.   Refer to Fact Pattern 22-1. According to the court’s ruling in Case 22.1, Utica Alloys, Inc. v. Alcoa, Inc., Beta is entitled to the proceeds from this sale and

a.   nothing more.
b.   the contract price without subtracting the market price for scrap.
c.   the difference between the contract price and the market price for processed scrap.
d.   the difference between the contract price and the market price for unprocessed scrap.

ANSWER:   D            PAGE:  437          TYPE:  N

8.   Refer to Fact Pattern 22-1. Under the court’s reasoning in Case 22.3, Utica Alloys, Inc. v. Alcoa, Inc., the reason for the measure of dam­ages awarded to Beta is that

a.   Acme was not the party who ended the deal—Beta was.
b.   Beta participated in the price review in good faith.
c.   Beta retrieved and sold scrap that Acme processed without paying for.
d.   Beta retrieved the processed scrap without paying for the processing.

ANSWER:   D            PAGE:  437          TYPE:  N



9.   Topps Publishing, Inc., contracts for a sale of textbooks to University Bookstores, Inc. (UBI). Value Shipping Company, the carrier, transports the books to Wit Warehouse Company. Topps’ right to stop delivery is lost when UBI’s rights to the goods are acknowledged by

a.   the appropriate government agency only.
b.   Value only.
c.   Wit only.
d.   Value or Wit.

     ANSWER:   D            PAGE:  438          TYPE:  N

10.  AAA Architects, Inc., and Best Office Supply Company contract for a sale of office furniture. AAA, which is insolvent, breaches the contract. Best can stop delivery of the goods in transit

a.   only if the quantity is at least a carload.
b.   only if the quantity is at least a planeload.
c.   only if the quantity is at least a truckload.
d.   regardless of the quantity.

     ANSWER:   D            PAGE:  438          TYPE:  +

11.  Hi-Tech Company contracts to sell fiber optic cable to Internet Services, Inc. Hi-Tech may bring an action to recover the purchase price and inci­dental damages if Internet

a.   accepts the cable and pays for it.
b.   accepts the cable but does not pay for it.
c.   rejects the cable.
d.   revokes acceptance of the cable.

     ANSWER:   B            PAGE:  438          TYPE:  +

12.  Pisa Pizza Company contracts to sell 1,000 cases of frozen pizzas to Quality Grocers, Inc., but refuses to deliver. Due to a spice shortage, Quality cannot obtain pizza elsewhere. Quality’s right to recover the goods from Pisa is the right of

a.   cover.
b.   replevin.
c.   repudiation.
d.   specific performance.

     ANSWER:   B            PAGE:  439          TYPE:  +

13.  Superior Furniture Company contracts to buy from Timber Products, Inc., a shipment of wood that Superior will use to make furniture. Timber refuses to deliver the wood. Superior can recover damages equal to the difference between the contract price and

a.   the cost to make the furniture.
b.   the cost to obtain the wood elsewhere.
c.   the profit that Superior would have made on the furniture.
d.   the sale price of the furniture.

     ANSWER:   B            PAGE:  439          TYPE:  N

14.  Omega Corporation contracts for a sale of water pumps to Pool & Spa, Inc. The pumps partly fail to conform to the contract. Pool & Spa

a.   may accept or reject the shipment in part or in whole.
b.   must accept the conforming part of the shipment.
c.   must reject the nonconforming part of the shipment.
d.   must reject the whole shipment.

     ANSWER:   A            PAGE:  440          TYPE:  N

15.  Hidebound Stores, Inc., rejects a shipment of leather goods that does not conform to its contract with International Cowhide Corporation (ICC), but is unable to obtain instructions from ICC. Hidebound may

a.   resell or return the goods only.
b.   resell or store the goods only.
c.   return or store the goods only.
d.   resell, return, or store the goods.

     ANSWER:   D            PAGE:  440          TYPE:  N

16.  A-1 BBQ, Inc., makes and sells barbecue grills to Big Mart, a retailer, which sells a grill to Carl, a consumer. A-1 and Big Mart include in their sales contracts a limitation on consequential damages for personal inju­ries from a breach of warranty. This is prima facie un­conscionable with respect to

a.   A-1 only.
b.   A-1 and Big Mart, but not Carl.
c.   Big Mart and Carl, but not A-1.
d.   Carl only.

     ANSWER:   D            PAGE:  443          TYPE:  N



Fact Pattern 22-2 (Questions 17–20 apply)
First State Bank issues a letter of credit in favor of Oboe Company, an American firm, to facilitate an international sales contract to buy resources from Lapland Mining, Ltd., a Finnish company.

17.  Refer to Fact Pattern 22-2. First State Bank must pay Lapland when Lapland

     a.   enters into the contract with Oboe.
     b.   verifies that Oboe has the money to pay for the purchase.
     c.   complies with the terms and conditions of the letter of credit.
     d.   none of the above.

     answer:   c            PAGE:  444          TYPE:  =

18.  Refer to Fact Pattern 22-2. First State Bank

a.   is responsible for making sure that the parties perform the contract.
     b.   will make payment once the transaction has been completed.
c.   will make payment when Lapland presents the proper documents.
d.   none of the above.

     answer:   c            PAGE:  444          TYPE:  =

19.  Refer to Fact Pattern 22-2. In a letter of credit, the ben­eficiary is

     a.   Lapland.
     b.   Oboe.
     c.   First State Bank.
     d.   none of the above.

     answer:   A            PAGE:  444          TYPE:  =

20.  Refer to Fact Pattern 22-2. To obtain payment, Lapland must comply with all of the requirements of the letter of credit

     a.   strictly.
     b.   substantially.
     c.   reasonably.
     d.   materially.

     answer:   A            PAGE:  444          TYPE:  =




Essay Questions

1.   Tech Electronics Corporation agrees to sell to Universal Retail Stores fifty televisions with LCD screens, to be delivered in five equal in­stallments. The first installment consists of nine TVs with LCD screens and one with a CRT screen. Can Universal cancel the whole contract?

ANSWER:   No. An installment contract is breached only if one or more noncon­forming installments sub­stantially im­pair the value of the whole contract. Also, a buyer cannot reject an install­ment unless a noncon­formity substan­tially impairs the value of the in­stallment and cannot be cured. The non­conforming tender here, under the cir­cumstances as stated, does not substan­tially impair the value of the install­ment or the whole contract and seems easily curable.

PAGES:    428–429                           type:  =

2.   Theatrical Supplies Company contracts to sell to United Costumes & Uniforms, Inc., seven hundred plastic masks at $1 each to be delivered by October 1. Theatrical knows that United will use the masks to make Halloween costumes. United usually makes $7,000 profit from the cos­tumes’ sale. Theatrical fails to deliver on October 1. United attempts to buy substitute masks, but must pay $1.20 for each and take delivery on October 15, cutting United’s sales in half. United sues Theatrical. What is the measure of recovery?

ANSWER:   The buyer should get the differ­ence between the substitute goods’ price and the contract price ($.20 per item). The buyer should also get any inciden­tal damages, and, because the seller knew the purpose of the purchase, the buyer should get consequential dam­ages, which could include the profits lost on the unmade sales.

PAGES:    439–440                           type:  =



Chapter 27


Checks, the Banking
System, and E-Money





N.B.:  TYPE indicates that a question is new, modified, or unchanged, as follows.

N   A question new to this edition of the Test Bank.
+   A question modified from the previous edition of the Test Bank,
=   A question included in the previous edition of the Test Bank.



TRUE/FALSE QUESTIONS

1.   UCC Articles 3 and 4 govern checks.

     ANSWER:   T            PAGE:  536          TYPE:  +

2.   Certified checks are instruments that have been accepted for payment by the institutions on which they are drawn.

     ANSWER:   T            PAGE:  537          TYPE:  =

3.   A bank is subject to a civil suit if its customer writes a bad check.

     ANSWER:   F            PAGE:  539          TYPE:  +

4.   Generally, a bank has no obligation to pay a customer’s overdrafts.

     ANSWER:   T            PAGE:  539          TYPE:  +

5.   When a check “bounces,” its holder can resubmit the check later, hop­ing that suf­ficient funds will be available.

     ANSWER:   T            PAGE:  539          TYPE:  +

6.   A bank has no right to charge a customer’s account for the amount of a stale check.

     answer:   f            PAGE:  540          TYPE:  N

7.   Banks are required to constantly verify the life and competence of their drawers.

     answer:   f            PAGE:  540          TYPE:  N

8.   A bank is obligated to pay an uncertified check presented less than six months from its date.

     ANSWER:   T            PAGE:  540          TYPE:  +

9.   An oral stop payment order is valid for thirty days.

     ANSWER:   F            PAGE:  540          TYPE:  +

10.  When a bank pays a check on which the drawer’s signature is forged, generally the customer suffers the loss.

     ANSWER:   F            PAGE:  541          TYPE:  N

11.  A forged signature has no legal effect as the signature of a drawer.

     ANSWER:   T            PAGE:  541          TYPE:  =

12.  A bank must always recredit a customer’s account when it pays a check with the cus­tomer’s forged signature.

     ANSWER:   F            PAGE:  541          TYPE:  +

13.  A bank is never obligated to recredit a customer’s account when it pays a check with the cus­tomer’s forged signature.

     ANSWER:   F            PAGE:  541          TYPE:  N

14.  When a bank pays a check on which the drawer’s signature is forged, generally the bank suffers the loss.

     ANSWER:   T            PAGE:  541          TYPE:  +

15.  A customer does not need to examine a bank statement and report his or her forged signa­ture to recover from the bank for the forgery.

     ANSWER:   F            PAGE:  543          TYPE:  N

16.  Generally, a cash deposit is not available for withdrawal until the next business day.

     ANSWER:   T            PAGE:  548          TYPE:  +

17.  The bank on which a check is drawn is the collecting bank.

     answer:   F            PAGE:  549          TYPE:  N

18.  Deferred posting is the process of dishonoring checks for insufficient funds.

     ANSWER:   F            PAGE: 550          TYPE:  =

19.  Money-service businesses have not been subject to regulation to the same extent as other financial service businesses.

     answer:   T            PAGE:  558          TYPE:  N

20.  Currently, it is not clear which, if any, laws apply to the security of e-money payment information.

     ANSWER:   T            PAGE:  558          TYPE:  N


MULTIPLE CHOICE QUESTIONS

1.   Standard Wholesalers, Inc., asks United Purchasing Com­pany to pay for goods with a certified check. A cer­tified check

a.   cannot be forged.
b.   does not discharge its drawer from liability.
c.   does not discharge prior indorsers from liability.
d.   is guaranteed by a bank.

ANSWER:   D            PAGE:  537          TYPE:  +

2.   Jay is the holder and payee of a check drawn by Karen on Local Bank. Jay takes the check to the bank to have it certified. After certification

a.   Karen is discharged on the check.
b.   Karen is primarily liable on the check.
c.   Local is discharged on the check.
d.   Local is secondarily liable on the check.

     answer:   A            PAGE:  537          TYPE:  N

3.   Pat, the manager of Quik Mart, deposits the store’s receipts in its account at Regional Bank. As to the receipts, the relation­ship between Quik Mart and the bank is

a.   attorney and client.
b.   creditor and debtor.
c.   guardian and ward.
d.   trustee and beneficiary.

ANSWER:   B            PAGE:  539          TYPE:  +

4.   Dan writes a check to Eve on his account at First State Bank. The bank dishonors the check even though Dan has sufficient funds in his ac­count. The bank is

a.   liable to Dan only.
b.   liable to Dan and Eve.
c.   liable to Eve only.
d.   not liable to Dan or Eve.

ANSWER:   A            PAGE:  539          TYPE:  N

5.   Dora writes a check for $100 drawn on Eastern Bank and presents it to Fast Cash, Inc., for payment. If the check is not backed by sufficient funds, Dora may be prosecuted for

a.  forgery.
b.  fraud.
c.  negligence.
d.  robbery.

ANSWER:  B            PAGE:  539          TYPE:  +

6.   On May 1, Ace Personnel, Inc., issues a payroll check to Barb drawn on its account at City Bank. On June 1, Ace receives its bank statement. On June 20, Barb indorses the check and cashes it at Downtown Finance Corporation. On July 1, Downtown transfers the check to EZ Collection Agency. On December 1, EZ presents the check to First National for pay­ment. A stale check is one that is presented for payment

a.   six months after issue.
b.   six months after the first indorsement.
c.   six weeks after issue.
d.   six weeks after the last statement.

ANSWER:   A            PAGE:  540          TYPE:  =

7.   Earl issues a check drawn on First National Bank to Good Office Supply to pay for six filing cabinets. Later, Earl discovers defects in the goods and orders First National to stop payment on the check. Earl does not renew the order, and the bank clears the check eight months later. The bank

a.   must recredit Earl’s account and substitute acceptable goods.
b.   must recredit Earl’s account or substitute acceptable goods.
c.   must substitute acceptable goods but not recredit Earl’s account.
d.   need not recredit Earl’s account or substitute acceptable goods.

ANSWER:   D            PAGE:  541          TYPE:  +

8.   First State Bank pays a check over the forged signature of the drawer, Greg, who is a First State customer. First State must recredit Greg’s account for the entire amount of the check if

a.   the amount of the check was less than $1,000.
b.   the amount of the check was more than $2,500.
c.   the bank failed to exercise ordinary care in paying the check.
d.   Greg’s negligence substantially contributed to the forg­ery.

ANSWER:   C            PAGE:  541          TYPE:  =

9.   Steve steals one of Tricia’s checks and forges her signature. Tricia’s bank, United Bank, pays the check. Tricia can recover from

a.   Steve, but not United Bank.
b.   United Bank, which cannot recover from Steve.
c.   United Bank, which can recover from Steve.
d.   no one.

ANSWER:   C            PAGE:  545          TYPE:  +

10.  Clyde issues a check payable to Discount Mart. Elle, Discount’s cashier, forges the store’s indorsement and deposits the check in her bank account. Clyde’s bank, First State Bank, pays the check. Clyde can recover from

a.   Elle, but not First State Bank.
b.   First State Bank, which cannot recover from Elle.
c.   First State Bank, which can recover from Elle.
d.   no one.

ANSWER:   C            PAGE:  546          TYPE:  +

11.  On Monday, Eve deposits in her account at First State Bank a local check for $500. After 5:00 p.m. on Friday, from these funds, Eve can withdraw no more than

     a.   $100.
     b.   $400.
     c.   $500.
     d.   $600.

     answer:   C            PAGE:  547          TYPE:  N

Fact Pattern 27-1  (Questions 12–14 apply)
Tom draws a check, on his account in State Bank in New York, payable to Digital Computers, Inc., in San Francisco. Digital deposits the check in its ac­count at First National Bank. First National deposits the check in the Federal Reserve Bank of San Francisco, which transfers it to the Federal Reserve Bank of New York. That Federal Reserve bank sends the check to State Bank.

12.  Refer to Fact Pattern 27-1.  Digital’s bank is

     a.   the cashing bank.
     b.   the depositary bank.
     c.   the intermediary bank.
     d.   the payor bank.

     answer:   B            PAGE:  549          TYPE:  N

13.  Refer to Fact Pattern 27-1. Tom’s bank is

     a.   the cashing bank.
     b.   the depositary bank.
     c.   the intermediary bank.
     d.   the payor bank.

     answer:   D            PAGE:  549          TYPE:  N

14.  Refer to Fact Pattern 27-1. When Digital’s bank received the check, it was required to pass it on

a.   before midnight of the next banking day.
b.   before midnight of the next day, whether or not it was a “banking” day.
c.   before noon of the next banking day.
d.   within five business days.

     answer:   A            PAGE:  550          TYPE:  N

15.  First National Bank receives a check drawn on the account of Rich Indus­tries, Inc., one of the bank’s customers, at 3 p.m. Friday. Sherry, the pre­senter of the check, is not one of the bank’s customers. The bank uses de­ferred posting with a 2 p.m. cutoff hour. If it decides to dis­honor the check, it must do so by midnight

a.   Saturday.
b.   Sunday.
c.   Monday.
d.   Tuesday.

     ANSWER:   D            PAGE:  551          TYPE:  =

16.  Bob’s debit card, issued by City Bank, is stolen and used without Bob’s permission. Bob tells City Bank within a day and a half. Bob may be required to pay no more than

a.   $5.
b.   $50.
c.   $500.
d.   $5,000.

     ANSWER:   B            PAGE:  556          TYPE:  N

17.  Dina’s debit card, issued by Eagle Bank, is stolen and used without Dina’s permission. Dina tells Eagle Bank within thirty days. Dina may be required to pay no more than

a.   $5.
b.   $50.
c.   $500.
d.   $5,000.

     ANSWER:   C            PAGE:  556          TYPE:  N

18.  Pete knowingly divulges to Media Exposure magazine information about Randy’s e-money payments to City Bank. The payments were in trans­mission to City Bank when Pete, without the consent of Randy or City Bank, discovered and revealed them. This may be a violation of

     a.   the Electronic Communications Privacy Act.
     b.   the Federal Reserve Board’s Regulation E.
c.   the Right to Financial Privacy Act.
d.   the Uniform Electronic Transactions Act.

     answer:   A            PAGE:  558          TYPE:  =

19.  AAA Financial Service is a money service business. Unlike a bank, AAA does not

a.   accept deposits.
     b.   cash checks.
c.   exchange foreign currency.
d.   issue money orders, traveler’s checks, and stored-value cards.

     answer:   A            PAGE:  558          TYPE:  N

20.  Liquid Cash Company is a brick-and-mortar money service business. Money Online, Inc., is an Internet-based money service. In those states that have adopted the Uniform Money Services Act, a state license is required to be obtained by

a.   Liquid Cash only.
     b.   Liquid Cash and Money Online.
c.   Money Online only.
d.   neither Liquid Cash nor Money Online.

     answer:   B            PAGE:  558          TYPE:  N


Essay Questions

1.   Joy steals a check from Kyle, forges his signature, and transfers the check to Loco Loans, Inc., for value. Unaware that the signature is not Kyle’s, Loco Loans presents the check to Metro Bank, the drawee, which cashes the check. Kyle discovers the forgery and insists that Metro recredit his account. Can Metro refuse? If not, from whom can the bank recover?

     ANSWER:     Metro cannot refuse to recredit Kyle’s account under the facts as stated in this problem. The general rule is that a bank must re­credit a customer’s account when it pays on the customer’s forged signature. Thus, the bank here cannot collect from its customer. Furthermore, a bank has no right to recover from a holder who, without knowledge, cashes a check bearing a forged drawer’s signa­ture, which appears to be the circumstance in this problem. Therefore, Metro cannot collect from Loco Loans either. The bank can collect the amount of the check from the thief who forged the signature, however. The bank’s best course of action is to charge Joy.

PAGES:    541–546                           type:  N

2.   Tom authorizes United Bank to make transfers from his account to make payments on his debt to Vic’s Auto Dealership, which sold Tom the car that serves as collateral for the debt. After three payments, Vic’s repos­sesses the car and refuses to return it. Tom phones the bank to stop the payments and follows up with a confirm­ing letter. The bank fails to stop the next two payments, and Vic’s refuses to refund anything. Can Tom get his money from the bank?

     ANSWER:     Yes. Under the Electronic Fund Transfer Act (EFTA), a financial institution is liable for failing to stop payment of a preautho­rized transfer from a cus­tomer’s ac­count when instructed to do so under the ac­count’s terms and conditions, which likely require that notice must precede a scheduled payment within a certain period of time. For other electronic transactions, reversal is generally not possible, however, be­cause of the instantaneous nature of the transactions, and the EFTA does not provide for their reversal.

PAGES:    556–557                           type:  =


Chapter 13


Sales, Leases, and E-Contracts



true/false questions

1.   Article 1 of the Uniform Commercial Code (UCC) governs contracts for sales of goods.

     answer:   F            PAGE:  387          TYPE:  N
Objective: AACSB Analytic   Skill Level: AICPA Legal

2.   The UCC governs sales of services.

     ANSWER:   F            PAGE:  388          TYPE:  N
Objective: AACSB Analytic   Skill Level: AICPA Legal

3.   In some instances, the UCC imposes different standards on merchants.

     answer:   T            PAGE:  389          TYPE:  +
Objective: AACSB Analytic   Skill Level: AICPA Legal

4.   Under the UCC, a party who sells one type of good will be considered a merchant for any other type of good that he or she may sell.

     answer:   f            PAGE:  389          TYPE:  N
Objective: AACSB Reflective Skill Level: AICPA Legal

5.   A lessor is one who sells the right to the possession and use of goods un­der a lease.

     answer:   T            PAGE:  391          TYPE:  N
Objective: AACSB Analytic   Skill Level: AICPA Legal

6.   According to the UCC, one or more open terms will always cause a sales or lease contract to fail for indefiniteness.

     answer:   F            PAGE:  392          TYPE:  N
Objective: AACSB Analytic   Skill Level: AICPA Legal

7. If a contract for a sale of goods does not include a price term, it is not enforceable.

     ANSWER:   F            PAGE:  392          TYPE:  N
Objective: AACSB Analytic   Skill Level: AICPA Legal

8.   Under the UCC, an agreement modifying a contract needs no considera­tion to be binding.

     ANSWER:   T            PAGE:  394          TYPE:  N
Objective: AACSB Analytic   Skill Level: AICPA Legal

9.   All oral contracts are enforceable under the UCC.

     ANSWER:   F            PAGE:  395          TYPE:  N
Objective: AACSB Analytic   Skill Level: AICPA Legal

10.  No oral contract is enforceable under the UCC.

     ANSWER:   F            PAGE:  395          TYPE:  N
Objective: AACSB Analytic   Skill Level: AICPA Legal

11.  A seller cannot exercise the right to cure once the time for performance under the contract has expired.

     answer:   t            PAGE:  399          TYPE:  N
Objective: AACSB Analytic   Skill Level: AICPA Legal

12.  Unless the parties agree otherwise, the buyer must pay at the time and place that goods are received.

     answer:   T            PAGE:  401          TYPE:  N
Objective: AACSB Analytic   Skill Level: AICPA Legal

13.  If a buyer breaches a contract and the seller resells the goods to another party, the seller cannot recover any loss from the breaching buyer.

     answer:   f            PAGE:  403          TYPE:  N
Objective: AACSB Analytic   Skill Level: AICPA Legal

14.  On a lessee’s insolvency, the lessor can stop delivery of the goods.

     answer:   T            PAGE:  403          TYPE:  N
Objective: AACSB Analytic   Skill Level: AICPA Legal


15.  Buying or leasing substitute goods for those that were due under a con­tract is known as obtaining a cure.

     ANSWER:   F            PAGE:  405          TYPE:  N
Objective: AACSB Analytic   Skill Level: AICPA Legal

16.  A lessor’s failure to make proper delivery of unique goods under a con­tract will not usu­ally give the lessee the right to specific performance.

     answer:   f            PAGE:  406          TYPE:  N
Objective: AACSB Analytic   Skill Level: AICPA Legal

17.  A buyer or lessee can obtain specific performance only when the goods that are the subject of a contract are not unique.

     ANSWER:   F            PAGE:  406          TYPE:  N
Objective: AACSB Analytic   Skill Level: AICPA Legal

18.  If goods fail to conform to a contract in any way, the buyer or lessee can­not make even a partial acceptance of the goods.

     ANSWER:   F            PAGE:  406          TYPE:  N
Objective: AACSB Analytic   Skill Level: AICPA Legal

19.  If a lessor’s tender of delivery fails to conform to a contract in any way, the lessee cannot accept the goods.

     ANSWER:   F            PAGE:  406          TYPE:  N
Objective: AACSB Analytic   Skill Level: AICPA Legal

20.  A buyer who accepts conforming goods cannot revoke the acceptance.

     ANSWER:   T            PAGE:  407          TYPE:  N
Objective: AACSB Analytic   Skill Level: AICPA Legal

21.  A buyer who accepts nonconforming goods cannot revoke the acceptance.

     ANSWER:   F            PAGE:  407          TYPE:  N
Objective: AACSB Analytic   Skill Level: AICPA Legal

22.  If the parties to a sales contract state that a certain remedy is exclusive, then it is the sole remedy.

     answer:   T            PAGE:  409          TYPE:  N
Objective: AACSB Analytic   Skill Level: AICPA Legal

23.  In sales law, a warranty is an assurance by one party of the existence of a fact on which the other party can rely.

     answer:   T            PAGE:  410          TYPE:  N
Objective: AACSB Analytic   Skill Level: AICPA Legal

24.  Advertisements can include express warranties.

     answer:   T            PAGE:  410          TYPE:  N
Objective: AACSB Analytic   Skill Level: AICPA Legal

25.  A seller must use words such as “warrant” or “guarantee” to make an express warranty.

     answer:   f            PAGE:  410          TYPE:  N
Objective: AACSB Analytic   Skill Level: AICPA Legal

26.  Only a statement made after a contract is entered into can be an express warranty.

     answer:   f            PAGE:  411          TYPE:  N
Objective: AACSB Analytic   Skill Level: AICPA Legal

27.  A product is unmerchantable if any accident could arise in connection with the goods.

     answer:   f            PAGE:  412          TYPE:  =
Objective: AACSB Analytic   Skill Level: AICPA Legal

28.  An implied warranty of merchantability arises in every sale or lease of goods by a non-merchant.

     answer:   F            PAGE:  412          TYPE:  N
Objective: AACSB Analytic   Skill Level: AICPA Legal

29.  Merchants are required to warrant that the goods they sell are fit for their ordinary purpose.

     answer:   t            PAGE:  412          TYPE:  N
Objective: AACSB Analytic   Skill Level: AICPA Legal

30.  A merchant can disclaim an implied warranty of merchantability.

     answer:   T            PAGE:  413          TYPE:  N
Objective: AACSB Analytic   Skill Level: AICPA Legal

31.  To prevent disputes and liability, an online contract offer should state its terms clearly and conspicuously.

     answer:   T            PAGE:  414          TYPE:  N
Objective: AACSB Analytic   Skill Level: AICPA Legal

32.  A click-on agreement is normally not enforced.

     answer:   F            PAGE:  415          TYPE:  N
Objective: AACSB Analytic   Skill Level: AICPA Legal

33.  Browse-wrap terms are arguably not enforceable.

     answer:   T            PAGE:  415          TYPE:  N
Objective: AACSB Analytic   Skill Level: AICPA Legal

34.  No contract may be denied legal effect solely because it is in electronic form.

     answer:   T            PAGE:  418          TYPE:  N
Objective: AACSB Analytic   Skill Level: AICPA Legal

35.  Under the Uniform Electronic Transactions Act (UETA), a contract is en­forceable even if it is solely in electronic form.

     answer:   T            PAGE:  418          TYPE:  N
Objective: AACSB Analytic   Skill Level: AICPA Legal


multiple choice questions

1.   Refining Corporation agrees to sell a storage tank on its premises to Stock Ranch Company. The tank can be removed by disconnecting a pipe. This deal is most likely

a.   a gift subject to the UCC.
b.   a lease subject to the UCC.
c.   a sale subject to the UCC.
d.   not subject to the UCC.

     ANSWER:   C            PAGE:  388          TYPE:  N
Objective: AACSB Reflective Skill Level: AICPA Legal


2.   Suzy pays Triple-A Office Supply Company $1,500 for a new laptop com­puter. According to the UCC, this is

a.   a gift.
b.   a lease.
c.   a sale.
d.   not a gift, a lease, or a sale.

     answer:   C            PAGE:  388          TYPE:  +
Objective: AACSB Reflective Skill Level: AICPA Legal

3.   Cleo sells kitchen appliances, and occasionally cleaning supplies, to per­sons who come into her store, Discount Appliances. One afternoon, Cleo sells a used display shelf to Earl. Under the UCC, Cleo is a merchant of

a.   cleaning supplies and kitchen appliances only.
b.   cleaning supplies, display shelves, and kitchen appliances.
c.   cleaning supplies only.
d.   kitchen appliances only.

     ANSWER:   A            PAGE:  389          TYPE:  N
Objective: AACSB Reflective Skill Level: AICPA Legal

4.   Metro Daily and New City Newsstand enter into a contract under which Metro agrees to deliver a certain quantity of newspapers to New City each day. The contract does not include a price term. In a suit between the par­ties over the price, a court will

a.   determine a reasonable price.
b.   impose the lowest market price.
c.   refuse to enforce the agreement.
d.   return the parties to the positions they held before the contract.

     ANSWER:   A            PAGE:  392          TYPE:  N
Objective: AACSB Reflective Skill Level: AICPA Legal

5.   International Gem Corporation agrees to sell Jewelry Outlets, Inc. (JOI), fifty new diamonds, but the contract does not specify a place of delivery. JOI is expected to pick up the goods. The place of delivery is

     a.   International’s place of business.
     b.   JOI’s place of business.
     c.   the Annual Gems and Jewels Convention.
     d.   the U.S. Postal Service office nearest to JOI’s place of business.

     answer:   A            PAGE:  392          TYPE:  N
Objective: AACSB Reflective Skill Level: AICPA Legal

6.   Regional, Inc., agrees to sell to Quantity Corporation a certain amount of goods to be delivered to Quantity’s warehouse, but no mention is made of the terms for payment. In general, the UCC requires that payment is due at

a.   a neutral place of business halfway between the parties’ locations
b.   a UCC-designated office.
c.   Regional’s place of shipping.
d.   Quantity’s place of receipt.

     ANSWER:   D            PAGE:  392          TYPE:  N
Objective: AACSB Reflective Skill Level: AICPA Legal

7.   Cathy and Dave sign a contract for a sale of goods in which they specify the quantity, price, payment terms, and delivery details. This contract is

a.   enforceable.
b.   unenforceable.
c.   void.
d.   voidable.

     ANSWER:   A            PAGE:  392          TYPE:  N
Objective: AACSB Reflective Skill Level: AICPA Legal

8.   Fine Coffee Company agrees to buy an unspecified quantity of coffee beans from Global AgriCorp. Global breaches the contract. Fine can most likely

a.   enforce the agreement to the extent of a reasonable quantity.
b.   enforce the agreement to the extent of Fine’s requirements.
c.   enforce the agreement to the extent of Global’s output of coffee beans.
d.   not enforce the agreement.

     ANSWER:   D            PAGE:  392          TYPE:  N
Objective: AACSB Reflective Skill Level: AICPA Legal

9.   All-Rite Clothiers, Inc., sells t-shirts to Brand Name Stores, Inc., under an existing con­tract. When textile costs increase, Brand agrees to a price increase, but later wants to cancel the con­tract. Brand may

a.   cancel the contract immediately.
b.   cancel the contract only after accepting a final shipment.
c.   cancel the contract only on reasonable notice.
d.   not cancel the contract.

     ANSWER:   D            PAGE:  394          TYPE:  N
Objective: AACSB Reflective Skill Level: AICPA Legal

10.  Kelly offers to buy cooking oil from Jim. Jim believes Kelly asks for 10,000 gallons and orally agrees to the sale. When the parties later dispute the deal in court, Jim’s claim of 10,000 gallons and Kelly’s testimony that she ordered only 1,000 gallons

a.   prevents the enforcement of any contract between these parties.
b.   supports an enforceable contract for 10,000 gallons.
c.   supports an enforceable contract for 5,500 gallons.
d.   supports an enforceable contract for 1,000 gallons.

     ANSWER:   D            PAGE:  397          TYPE:  N
Objective: AACSB Reflective Skill Level: AICPA Legal

11.  Recreational Supplies, Inc. (RSI), and Sam, the owner of a Tourist Time shop, orally agree to a sale of beach balls and seashells for $1,000. Sam gives RSI a check for $400 as a partial payment. This contract is

a.   enforceable to the extent of $400.
b.   fully enforceable because it is for specially made goods.
c.   fully enforceable because it is oral.
d.   not enforceable.

     ANSWER:   A            PAGE:  398          TYPE:  N
Objective: AACSB Reflective Skill Level: AICPA Legal

12.  Price-Cut Discount Stores are open to consumers. The UCC requirement of good faith imposes

     a.   a higher duty on consumers than Price-Cut.
     b.   a higher duty on Price-Cut than on consumers.
     c.   no duty on either Price-Cut or consumers.
     d.   the same duty on Price-Cut and consumers.

     answer:   B            PAGE:  398          TYPE:  N
Objective: AACSB Reflective Skill Level: AICPA Legal

13.  Superb Resources, Inc., sells unprocessed minerals to commercial proc­essors in Texas. With regard to the UCC’s good faith requirement, Superb can

a.   avoid it only by a conspicuous written disclaimer.
b.   avoid it only by oral disclaimer.
c.   avoid it with or without a disclaimer.
d.   not disclaim it.

     answer:   D            PAGE:  398          TYPE:  N
Objective: AACSB Reflective Skill Level: AICPA Legal

14.  Pep Paints agrees to sell to Quality Painters Grade A-1 latex outdoor paint to be delivered May 8. On May 7, Pep tenders Grade B-2 paint. Quality re­jects the Grade B-2 paint. Two days later, Pep tenders Grade C-3 paint with an offer of a price allowance. Pep has

a.   additional, unlimited time to cure.
b.   a reasonable, additional time to cure.
c.   one more day to cure.
d.   no more time to cure.

     answer:   D            PAGE:  399          TYPE:  N
Objective: AACSB Reflective Skill Level: AICPA Legal

15.  Doctors Clinic orders 1,000 bandages from Emergency Supplies Company but fails to specify the sizes. The bandages are delivered in an assortment of sizes. Doctors Clinic may

a.   accept the bandages “as is” only.
b.   accept the bandages “as is” or reject the entire shipment only.
c.   accept only the bandages that it wants and reject the rest.
d.   reject the entire shipment only.

     ANSWER:   C            PAGE:  402          TYPE:  N
Objective: AACSB Reflective Skill Level: AICPA Legal

16.  AAA Architects, Inc., and Best Office Supply Company contract for a sale of office furniture. AAA, which is insolvent, breaches the contract. Best can refuse to deliver the goods

a.   under no circumstances.
b.   unless AAA agrees to pay a higher price for the goods.
c.   unless AAA pays in cash.
d.   unless a court guarantees AAA’s payment for the goods.

     ANSWER:   C            PAGE:  402          TYPE:  N
Objective: AACSB Reflective Skill Level: AICPA Legal

17.  Omega Corporation contracts for a sale of water pumps to Pool & Spa, Inc. The pumps partly fail to conform to the contract. Pool & Spa

a.   may accept or reject the shipment in part or in whole.
b.   must accept the conforming part of the shipment.
c.   must reject the nonconforming part of the shipment.
d.   must reject the whole shipment.

     ANSWER:   A            PAGE:  402          TYPE:  N
Objective: AACSB Reflective Skill Level: AICPA Legal

18.  Glad Chair Company contracts to deliver 100 chairs to Home Furnishings Store on May 1. Glad tells Home on April 15 that it is canceling the con­tract be­cause its workers have gone on strike. Home may

a.   await Glad’s performance, sue Glad, or suspend its own performance.
b.   only await Glad’s performance for a commercially reasonable time.
c.   only sue Glad for breach of contract.
d.   only suspend its own performance.

     answer:   A            PAGE:  402          TYPE:  N
Objective: AACSB Reflective Skill Level: AICPA Legal

19.  Superior Furniture Company contracts to buy from Timber Products, Inc., a shipment of wood that Superior will use to make furniture. Timber refuses to deliver the wood. Superior can recover damages equal to the difference between the contract price and

a.   the cost to make the furniture.
b.   the cost to obtain the wood elsewhere.
c.   the profit that Superior would have made on the furniture.
d.   the sale price of the furniture.

     ANSWER:   B            PAGE:  405          TYPE:  N
Objective: AACSB Reflective Skill Level: AICPA Legal

20.  Drew contracts for a sale of tools to Engineering Company (EC). If the tools fail to conform to the contract, EC

a.   may accept or reject the shipment in part or in whole.
b.   must accept the shipment.
c.   must reject the shipment.
d.   must renegotiate the contract.

     ANSWER:   A            PAGE:  407          TYPE:  N
Objective: AACSB Reflective Skill Level: AICPA Legal

21.  Digital Video Company contracts to sell DVD players to E-lectric Stores, Inc. Digital Video ships nonconforming goods, which E-lectric accepts. E-lectric can recover damages equal to the difference between

a.   E-lectric’s sale price and the contract price.
b.   the contract price and the value of the goods as promised.
c.   the value of the goods as promised and their value as accepted.
d.   the value of the goods as accepted and E-lectric’s sale price.

     ANSWER:   C            PAGE:  407          TYPE:  N
Objective: AACSB Reflective Skill Level: AICPA Legal

22.  Olivia visits Perfectly Fine Cars, an auto dealer, and says she wants a car that gets at least thirty miles to the gallon. Quinn, a salesperson, recom­mends a Roadster, which he says gets “at least forty miles to the gallon.” This is

     a.   an express warranty.
     b.   an implied warranty of merchantability.
     c.   an implied warranty of fitness for a particular purpose.
     d.   a warranty of title.

     answer:   A            PAGE:  410          TYPE:  N
Objective: AACSB Reflective Skill Level: AICPA Legal

23.  Pat, a salesperson for Quality Textiles, Inc., shows Rosa, a fabric buyer for Style Clothing Company, samples of cloth, stating that any ship­ment will match the samples. This statement is

a.   an express warranty.
b.   an implied warranty.
c.   a warranty of title.
d.   puffery.

     answer:   A            PAGE:  410          TYPE:  +
Objective: AACSB Reflective Skill Level: AICPA Legal

Fact Pattern 13-1 (Questions 24–25 apply)
Ann, a representative of Best Concrete Products, Inc. assures City Construction Company (CCC) that Best’s cement will not crack within a certain range of temperatures. CCC uses Best’s product. When cracks develop within the stated temperature range, CCC files a suit against Best.

24.  Refer to Fact Pattern 13-1. The court is most likely to rule in favor of

a.   Best, because Ann’s statement was an expression of opinion.
b.   Best, because CCC chose Best’s product voluntarily.
c.   CCC, because Ann’s statement was an express warranty.
d.   CCC, because Best’s product is not fit for its purpose.

     answer:   C            PAGE:  410          TYPE:  N
Objective: AACSB Reflective Skill Level: AICPA Legal


25.  Refer to Fact Pattern 13-1. Suppose that the court rules against Best. The manufacturer might have avoided that result by

a.   making its cement fit for its particular purpose.
b.   making its cement merchantable for its intended use.
c.   not allowing Ann to express an opinion.
d.   not permitting Ann to make an express warranty.

     answer:   D            PAGE:  411          TYPE:  N
Objective: AACSB Reflective Skill Level: AICPA Legal

26.  Eve, a salesperson for Finest Cars & Trucks, Inc., tells Gus, “This is the best car I’ve ever seen—and I’ve seen a lot of cars.” This statement is

a.   an express warranty.
b.   an implied warranty.
c.   a warranty of title.
d.   puffing.

     answer:   D            PAGE:  411          TYPE:  N
Objective: AACSB Reflective Skill Level: AICPA Legal

27.  AAA Appliance salespersons tell potential customers that AAA prod­ucts are “idiot-proof.” This is

a.   an express warranty.
b.   an implied warranty.
c.   a warranty of title.
d.   puffing.

     answer:   D            PAGE:  411          TYPE:  N
Objective: AACSB Reflective Skill Level: AICPA Legal

28.  Allen buys a new sport utility vehicle (SUV) from Best Cars & Trucks, Inc. The most important factor in determining whether an ex­press war­ranty is created is whether

     a.   Allen expresses to Best what he wants warranted.
     b.   Allen’s desire for the SUV becomes part of his motivation to deal.
     c.   Best expresses to Allen what it expects of its customers.
     d.   Best’s promise becomes part of the basis of the bargain.

     answer:   D            PAGE:  411          TYPE:  N
Objective: AACSB Reflective Skill Level: AICPA Legal


29.  Neil goes to Oil Shop to change the oil in his car. Pat, the service tech­ni­cian, learns that Neil plans to take a trip and advises the use of a certain type of oil. The oil breaks down during the trip, damaging the car. Most likely, Oil Shop breached

     a.   an express warranty.
     b.   an implied warranty of fitness for a particular purpose.
     c.   an implied warranty of merchantability.
     d.   no warranty.

     answer:   B            PAGE:  412          TYPE:  N
Objective: AACSB Reflective Skill Level: AICPA Legal

30.  Great Goods, Inc., sells two hundred MP3 players to Happy Stores, Inc. To avoid liability for most implied warranties, Great Goods should state in writing that the players are sold

a.   as is.
b.   by a merchant.
c.   in perfect condition.
d.   with no known defects.

     answer:   A            PAGE:  413          TYPE:  +
Objective: AACSB Reflective Skill Level: AICPA Legal

31.  Don buys from E-Stuf, Inc., software that includes an on-screen click-on “I agree to the terms” box. A dispute arises, and E-Stuf files a suit against Don. The court will most likely enforce the terms if Don used the software

     a.   after Don had an opportunity to read the terms.
     b.   before Don had an opportunity to read the terms.
     c.   only after Don actually read the terms.
     d.   whether or not Don read the terms.

     answer:   A            PAGE:  415          TYPE:  N
Objective: AACSB Technology Skill Level: AICPA Legal

32.  Cody and Dana engage in a transaction that involves e-documents. The E-SIGN Act applies if those documents include

     a.   court papers.
     b.   evictions.
c.   foreclosures.
     d.   none of the above.

     answer:   D            PAGE:  418          TYPE:  N
Objective: AACSB Reflective Skill Level: AICPA Legal

33.  Ann, a seller, and Bill, a buyer, make a deal over the Internet that in­volves e-signatures. Under the Electronic Signatures in Global and National Commerce (E-SIGN) Act, for the e-signatures to be enforceable

a.   both parties must have agreed to use e-signatures.
     b.   neither party must have agreed to use e-signatures.
     c.   only Ann must have agreed to use an e-signature.
     d.   only Bill must have agreed to use an e-signature.

     answer:   A            PAGE:  418          TYPE:  N
Objective: AACSB Reflective Skill Level: AICPA Legal

34.  Mary and Nick make a deal that comes under the UETA. Under the UETA, “information that is inscribed on a tangible medium or that is stored in an electronic or other medium and is retrievable in perceivable form” is

     a.   an e-document.
     b.   an e-signature.
c.   an e-transaction.
     d.   a record.

     answer:   D            PAGE:  418          TYPE:  N
Objective: AACSB Reflective Skill Level: AICPA Legal

35.  Kay and Lucy enter into a contract that falls within the provisions of the Uniform Electronic Transactions Act (UETA). Under the UETA, “an elec­tronic sound, symbol, or process attached to or logically associated with a record and executed or adopted by a person with the intent to sign the re­cord” is

     a.   an e-document.
     b.   an e-signature.
c.   an e-transaction.
     d.   a record.

     answer:   B            PAGE:  418          TYPE:  N
Objective: AACSB Reflective Skill Level: AICPA Legal


Essay Questions

1.   Great Beverages Company (GBC) often sells coffee, tea, and related prod­ucts to Hava Java, Inc. (HJI), which owns and operates a chain of coffee shops. Over the phone, representatives of the two firms negotiate a sale of 144 pounds of a certain kind of coffee bean. GBC sends a letter to HJI de­tailing the terms and ten days later ships the beans. Is there an enforce­able con­tract between them? Explain.

ANSWER:   There is an enforceable contract between these parties. In a transaction between mer­chants, the re­quirement of a writing is satisfied if one of them sends to the other a signed written confirmation that indi­cates the terms of the agree­ment, and the merchant receiving it has rea­son to know of its contents. If the merchant who receives it does not object in writing within ten days after re­ceipt, the writing will be en­forceable against the merchant even though he or she has not signed anything.

PAGE:     395                               type:  N
  Objective: AACSB Reflective Skill Level: AICPA Decision
                                  Modeling

2.   Alpha Electronic Components, Inc., ships 100,000 silicon chips to Beta E-Products Corporation. The chips arrive a week early, on a Friday that is extremely busy on Beta’s receiving dock. Beta’s dockworkers check the bill of lading against the quantity marked on the boxes, but do not exam­ine the chips. The chips are then put in the back of the warehouse until needed in the plant. The next week, when the chips are sent to the plant and unpacked, Beta’s plant manager discovers that the quality is less than that stated in the parties’ contract. Beta contacts Alpha to inform it of the defect. Does Beta have any remedies? If so, what are they? If not, why not?

ANSWER:   Beta has remedies in this situation: it can reject the goods and then obtain cover or cancel the contract. A buyer has the right to in­spect the goods that it receives, if that inspection is done within a reason­able time of the receipt. Here, although the inspection occurred a few days after the delivery, the delivery was early and happened on a day that was busy for Beta, both reasons that would justify a delayed examination of the goods. A buyer can reject nonconforming goods within a reason­able time if the seller is notified seasonably—that is, in a timely fashion. In this problem, Beta notified Alpha promptly. Even if Beta were consid­ered to have accepted the goods after its quick inspection the week before, it could revoke that acceptance. A buyer who accepts goods may revoke accep­tance within a reasonable time if the acceptance was reasonably in­duced by the difficulty of discovering the defect.

PAGES:    402 & 405–409                     type:  =
  Objective: AACSB Reflective Skill Level: AICPA Decision
                                  Modeling


Chapter 44


Consumer Law





N.B.:  TYPE indicates that a question is new, modified, or unchanged, as follows.

N   A question new to this edition of the Test Bank.
+   A question modified from the previous edition of the Test Bank,
=   A question included in the previous edition of the Test Bank.




TRUE/FALSE QUESTIONS


1.   The test for whether an ad is deceptive is whether a reason­able con­sumer would be deceived.

     ANSWER:   T            PAGE:  888          TYPE:  =

2.   Puffing is prohibited.

     ANSWER:   F            PAGE:  888          TYPE:  +

3.   An ad that contains a “half truth”—information that is true but incom­plete—is not deceptive.

     ANSWER:   F            PAGE:  888          TYPE:  +

4.   A company charged with deceptive advertising must go through a hear­ing to resolve the complaint.

     ANSWER:   F            PAGE:  889          TYPE:  +

5.   Product labels do not need to be accurate if they use easily un­derstood words.

     ANSWER:   F            PAGE:  890          TYPE:  =

6.   Labels on packages must use words that can be explained by the ordi­nary marketing executive.

     ANSWER:   F            PAGE:  890          TYPE:  N

7.   Counteradvertising is prohibited.

     ANSWER:   F            PAGE:  890          TYPE:  N

8.   Counteradvertising is advertising that counters a competitor’s claims.

     ANSWER:   F            PAGE:  890          TYPE:  =

9.   When a consumer cancels an order, a mail-order merchant must issue a refund within a specific period of time.

     ANSWER:   T            PAGE:  891          TYPE:  =

10. Online sales are not yet subject to state or federal consumer laws.

     ANSWER:   F            PAGE:  891          TYPE:  =

11.       Regulation Z spells out what information must be disclosed in transac­tions subject to the Truth-in-Lending Act.

     ANSWER:   T            PAGE:  892          TYPE:  =

12. A sale between two consumers is not subject to the Truth-in-Lending Act.

     ANSWER:   T            PAGE:  892          TYPE:  +

13. A loan between two consumers is subject to the Truth-in-Lending Act.

     ANSWER:   F            PAGE:  892          TYPE:  N

14. The Equal Credit Opportunity Act prohibits the denial of credit on the basis of an individual’s financial status.

     ANSWER:   F            PAGE:  892          TYPE:  N

15.  An individual is not liable for unauthorized charges on a credit card that he or she did not solicit.

     ANSWER:   T            PAGE:  892          TYPE:  =

16.       A credit agency that negligently fails to delete inaccu­rate information from a person’s credit report is liable.

     ANSWER:   T            PAGE:  893          TYPE:  =

17.  Creditors can contact a debtor only at the debtor’s place of employment.

     ANSWER:   F            PAGE:  895          TYPE:  N

18.  A creditor may contact at any time a debtor who has refused to pay a debt.

     ANSWER:   F            PAGE:  895          TYPE:  +

19. The Consumer Product Safety Commission can ban a product that poses an “unreasonable risk” to consumers.

     ANSWER:   T            PAGE:  897          TYPE:  =

20. All consumer protection laws are federal.

     ANSWER:   F            PAGE:  898          TYPE:  N


MULTIPLE CHOICE QUESTIONS

1.   Tech Company’s ad states that “if you aren’t using a Tech computer, you aren’t using the best.” The Federal Trade Com­mission would consider this ad

a.   false and misleading.
b.   false only.
c.   misleading only.
d.   neither false nor misleading.

     answer:   D            PAGE:  888          TYPE:  =

2.   A-One GPS Company’s ad states that its product is “the finest that money can buy.” Because of this ad, the Federal Trade Com­mission may

a.   issue a cease-and-desist order only.
b.   issue a cease-and-desist order or require counteradvertising.
c.   neither issue a cease-and-desist order nor require counteradvertising.
d.   require counteradvertising only.

     answer:   C            PAGE:  889          TYPE:  N

3.   A+ Goods, Inc., engages in de­ceptive advertising when it markets its product Best Cleaner as able to kill germs over long periods of time. In an action against A+ regarding Best, A+ is ordered to stop its false advertis­ing of Best and other products. This is

     a.   a counteradvertising order.
     b.   a multiple product order.
     c.   bait-and-switch advertising.
     d.   excessive abuse of authority.

     ANSWER:   B            PAGE:  890          TYPE:  =

Fact Pattern 44-1 (Questions 4–5 apply)
Phone Sales, Inc., solicits business from con­sumers over the phone. Quinn, a consumer, objects to being solicited in this way.

4.   Refer to Fact Pattern 44-1. Quinn may suc­ceed in a suit against Phone Sales if the firm

a.   discloses to consumers that its calls are sales calls.
b.   informs consumers of restrictions on using its products.
c.   tells consumers that all sales are final.
d.   uses an automatic dialing system to make its calls.

     ANSWER:   D            PAGE:  890          TYPE:  N

5.   Refer to Fact Pattern 44-1. Suppose that Phone Sales uses a prerecorded voice to solicit business over the phone. Rita objects. In a suit against Phone Sales, she may recover

a.   nothing.
b.   only an injunction against any future calls.
c.   up to $500 in damages for each objectionable call.
d.   up to treble the amount of any monetary loss.

     ANSWER:   C            PAGE:  890          TYPE:  +

6.   Best Goods Corporation makes and sells consumer goods. Best Goods’ product labels must identify

     a.   the address to which complaints can be directed.
b.   the federal agency that inspected the product.
     c.   the manufacturer.
     d.   the percentage of the product that consists of artificial substances.

     ANSWER:   C            PAGE:  890          TYPE:  +

7.   Crunchy Corporation makes and sells ce­real. Crunchy’s packaging must include

     a.   neither the identity of the product nor the number of servings.
     b.   the identity of the product and the number of servings.
c.   the identity of the product only.
     d.   the number of servings only.

     ANSWER:   B            PAGE:  891          TYPE:  +

8.   Ron signs a contract with Sam, a door-to-door salesperson for Tutors, Inc., to buy a foreign-language course. To cancel the contract, Ron has up to

a.   three days.
b.   thirty days.
c.   sixty days.
d.   ninety days.

     ANSWER:   A            PAGE:  891          TYPE:  =

9.   Friendly Loan Company extends credit in the ordinary course of its busi­ness. Under the Truth-in-Lending Act, Friendly must inform potential borrowers of

a.   credit terms offered by other lenders and its own credit terms.
b.   only credit terms offered by other lenders.
c.   only its own credit terms.
d.   only the credit terms that will convince borrowers to “close the deal.”

     ANSWER:   C            PAGE:  892          TYPE:  =

10.  Brad borrows $20,000 from City Bank to repair his home and to buy a car. Brad buys a stereo from Delta Store in a transaction financed by Delta. If these parties are subject to the Truth-in-Lending Act, Regulation Z applies to

a.   the car loan only.
b.   the home repair loan only.
c.   the retail installment sale only.
d.   the car loan, the home repair loan, and the retail installment sale.

     ANSWER:   D            PAGE:  892          TYPE:  =


11.  Owen signs an installment contract with Pixel Video Store to finance the purchase of a new TV for $4,999. This transaction is sub­ject to

a.   no federal law.
b.   the Fair Credit Reporting Act.
c.   the Telecommunications Act.
d.   the Truth-in-Lending Act.

     ANSWER:   D            PAGE:  892          TYPE:  +

12.  Eve borrows $40,000 from First National Bank to add a room to her home. This transaction is subject to

a.   the Consumer Leasing Act.
b.   the Magnuson-Moss Warranty Act.
c.   the Truth-in-Lending Act.
d.   the Uniform Commercial Code.

     ANSWER:   C            PAGE:  892          TYPE:  +

13.  Consumer Finance Corporation (CFC) extends credit to consumers. CFC is subject to the Equal Credit Opportunity Act, which prohibits credit discrimination based on

a.   educational achievement.
b.   income.
c.   marital status.
d.   unfavorable credit reports.

     ANSWER:   C            PAGE:  892          TYPE:  =

14.  Kent receives an unsolicited credit card in the mail and tosses it on his desk. Without Kent’s permission, his roommate uses the card to buy a new per­sonal computer for $1,000. Kent is

a.   liable for $1,000.
b.   liable for $500.
c.   liable for $50.
d.   not liable for any amount.

     ANSWER:   D            PAGE:  892          TYPE:  =


15.  Quik Collection Agency calls Pat several times a day, and some­times in the middle of the night, about an overdue bill that Regal Sporting Goods turned over to Quik for collection. This is a violation of

a.   no federal law.
b.   the Fair and Accurate Credit Transactions Act.
c.   the Fair Debt Collection Practices Act.
d.   the Truth-in-Lending Act.

     ANSWER:   C            PAGE:  895          TYPE:  N

16.  Furniture Depot sells Gail a bedroom suite on credit. Gail fails to make the scheduled payments for six months. Furniture Depot sends her a let­ter, asking for immediate payment. This is a violation of

a.   no federal law.
b.   the Fair and Accurate Credit Transactions Act.
c.   the Fair Debt Collection Practices Act.
d.   the Truth-in-Lending Act.

     ANSWER:   A            PAGE:  895          TYPE:  N

17.  The credit department of Mega-Mart often calls Nora at work about an overdue bill. Nora’s employer objects. This is a viola­tion of

a.   no federal law.
b.   the Fair and Accurate Credit Transactions Act.
c.   the Fair Debt Collection Practices Act.
d.   the Truth-in-Lending Act.

     ANSWER:   C            PAGE:  895          TYPE:  N

18.  Corner Market sells groceries. Delta Food & Drug Store sells groceries and fills prescriptions. The party with the chief responsibility to prevent unsafe food and drugs from being sold is

a.   Corner Market and Delta Food & Drug Store.
b.   Delta Food & Drug Store only.
c.   the Federal Trade Commission.
d.   the Food and Drug Administration.

     answer:   D            PAGE:  896          TYPE:  N


19.  Steel Tool Company makes and sells tools. One of the tools is believed to be haz­ardous. The appropriate government agency may require Steel to

a.   export the tool and sell it only abroad.
b.   increase the price to cover the cost of any injuries or damage.
c.   reduce the price to indicate the hazard to consumers.
d.   remove the tool from the market.

     answer:   D            PAGE:  897          TYPE:  N

20.  American Sales Corporation does business in all fifty states and is sub­ject to state consumer protection laws in those states in which they ex­ist. Those laws are

a.   enacted by state legislatures.
b.   enforced by the Consumer Product Safety Commission.
c.   required by the National Conference of Commissioners on Uniform State Laws.
d.   reviewed by the Federal Trade Commission.

     answer:   A            PAGE:  898          TYPE:  =


Essay Questions

1.   Alpha Corporation makes batteries for motor vehicles. The Federal Trade Commission (FTC) learns that Beta Automotive Stores, a retail company that sells Alpha’s batteries, engages in deceptive advertising practices. What actions can the FTC take against Beta?

     ANSWER:     The Federal Trade Commission (FTC) can investigate the prob­lem, and if, after the investigation, it believes that Beta has engaged in deceptive advertising, it can send a formal complaint to the al­leged of­fender.  Beta may agree to a settlement.  If not, the FTC can conduct a hearing before an administrative law judge (ALJ).  If the FTC succeeds in proving that Beta’s ads are deceptive, it can issue a cease-and-desist order requiring the advertising to stop.  It might also im­pose a coun­teradvertising sanction, requiring Beta to issue new ads—in print, on radio, and on television—to inform the public about the earlier misinformation.

PAGES:    889–890                      type:     N

2.   Eats Company wants to sell its candy in a normal-sized package la­beled “Gigantic Size.” Fine Fabrics, Inc., wants to advertise its sweaters as hav­ing “That Wool Feel,” but does not want to specify on labels that the sweat­ers are 100 percent polyester. What stops these firms from market­ing their products as they would like?

     ANSWER:   There are a number of federal and state laws that deal spe­cifically with in­formation given on la­bels and pack­ages, including the Fair Packaging and La­beling Act of 1966 and the Wool Prod­ucts Labeling Act of 1939. In general, the information on such labels must be truthful. There are also other state and federal laws that prohibit unfair and decep­tive advertising, including labels. These laws include the Federal Trade Commission Act of 1914.

     PAGE:     890                               TYPE:  =
 Chapter 20


The Formation of
Sales and Lease Contracts
multiple choice questions

A1.      Bild-Rite, Inc., is a Colorado-based firm that does business with clients throughout North America. Bild-Rite oversees construction projects, and buys and sells commercial buildings, undeveloped land, and construction supplies and other goods. Bild-Rite has had to deal with work-site theft and vandalism. With respect to these circumstances, the Uniform Commercial Code (UCC) provides a framework for
a.   commercial transactions for the sale of and payment for goods.
b.         international construction contracts.
c.         domestic and foreign transactions in real estate.
d.         prosecuting crimes against business interests.

B1.      Stardust Coffee Company is a Texas-based firm that does business throughout the world. Stardust manages retail and wholesale operations, buys and sells commercial venues, undeveloped land, and coffee beans, and other goods. Stardust has had to deal with employee and customer theft. With respect to these circumstances, the Uniform Commercial Code (UCC) provides a framework for
a.   commercial transactions for the sale of and payment for goods.
b.         international distribution agreements.
c.         domestic and foreign transactions in real estate.
d.         prosecuting crimes against business interests.
A2.      Omni Corporation is a Pennsylvania-based firm that does business throughout the United States. With respect to this circumstance, the UCC has been adopted by, and applies in,
a.   a few of the states.
b.         all of the states, in whole or in part.
c.         half of the states.
d.         none of the states, to date.
B2.      High n’ Mighty Mart, Inc., is an Illinois-based firm that does business throughout the United States. With respect to this circumstance, the UCC has been adopted by, and applies in,
a.         all of the states, in whole or in part.
b.         most of the states on the Atlantic and Pacific coasts.
c.         none of the states, to date.
d.   only the states on the Mississippi, Missouri, and Ohio Rivers.

A3.      Over the course of a year, Retail Marketers, Inc., sells goods from its inventory and one of its warehouses. In exchange, Retail receives checks and other items that substitute for cash, which Retail uses to repay a loan from Savings Bank. Article 2 of the UCC governs
a.         the checks.
b.         the payment of the loan.
c.   the sale of the buildings.
d.         the sale of the goods.


B3.      Over the course of a year, Real Deal Corporation sells appliances to customers to whom it extends credit. Real Deal orders the appliances from Superior Appliance Company’s warehouse, from which the items are shipped via common carrier to Real Deal’s customers. Article 2 of the UCC governs
a.         all of the parties’ sales of the goods.
b.   Real Deal’s extension of credit.
c.         Superior’s storage of the goods.
d.         the common carrier’s delivery of the goods.
A4.  Nemo pays Office Supply Company $1,500 for a laptop com­puter. Under the UCC, this is
a.   a gift.
b.   a lease.
c.   a sale.
d.   not a gift, a lease, or a sale.

B4.      Expert Stitching Corporation enters into a contract to sell denim clothing to Fine Fashion Company, which in turn sells a pair of jeans to Grady, a consumer. In contrast to standards that apply to consumers, the UCC imposes on merchants
a.         less strict legal standards.
b.         special business standards.
c.   stricter ethical standards.
d.         the same overall standards.
A5.      In a dispute over a sale involving a bicycle, Dain argues that as to this deal Elle’s Hobby Shop, where Dain bought the bike, is a merchant. A court may determine whether Elle’s is a merchant by assessing whether
a.         it has sold any bikes within the last year.
b.         it holds itself out by occupation as having knowledge or skill unique to the bike in the transaction.
c.         its owner enjoys biking.
d.         it subscribes to Bike, a biweekly trade magazine.
B5.      Rikki and Sid enter into a sales contract. With respect to the specific contractual provisions set out in the UCC, Rikki and Sid may
a.         agree to different terms only to a reasonable extent.
b.         agree to different terms unless they “get caught.”
c.         agree to whatever terms they wish.
d.         not agree to different terms.
A6.      Cleo sells kitchen appliances, and occasionally cleaning supplies, to per­sons who come into her store, Discount Appliances. One afternoon, Cleo sells a used display shelf to Earl. Under the UCC, Cleo is a merchant of
a.         cleaning supplies and kitchen appliances only.
b.         cleaning supplies, display shelves, and kitchen appliances.
c.         cleaning supplies only.
d.         kitchen appliances only.
B6.      Excel Autos & Trucks, Inc., contracts to sell five trucks to First Leasing Corporation, which contracts to lease the trucks to General Delivery Company. Article 2A of the UCC applies to
a.         neither the lease nor the sale.
b.         the lease and the sale.
c.         the lease only.
d.         the sale only.
A7.      Rally Corporation enters into a contract to sell ski gear to SnoSportz Company, which sells a pair of the skis to Tyra, a consumer, who later sells them to Uli, another consumer. Article 2 of the UCC applies to  the sales transactions between
a.         all of the buyers and sellers.
b.         Rally and SnoSportz only.
c.   SnoSportz and Tyra only.
d.         Tyra and Uli only.


B7.  American Products Company and Best Manufacturing, Inc. (BMI), enter into a contract for the sale of a certain quantity of machine parts, with BMI to determine the price. The price must be fixed according to the con­cept of
a.   good faith.
b.         square dealing.
c.         the mere image rule.
d.         unconscionability.
A8.      Curtis enters into a contract with Drive-Away Lease Company for a three-year lease of a car. This contract is subject to
a.         Article 2 of UCC.
b.   Article 2A of the UCC.
c.         Article 11 of the CISG.
d.         the common law only.

B8.      Cathy and Dave sign a contract for a sale of goods. Cathy is to set the price for the goods at the time of delivery, but on delivery, refuses to do so. Dave may only
a.   fix a reasonable price.
b.         fix a reasonable price or treat the contract as canceled.
c.         treat the contract as canceled.
d.         wait for Cathy to set the price.

A9.      Standard Office Corporation pays Tech Products $1,000 to use a Tech com­puter for a month. For purposes of the UCC, this is
a.   a bailment.
b.   a consignment.
c.   a lease.
d.   a sale.

B9.      Variety Goods, Inc., and World Sales Corporation enter into a contract that does not specify the payment terms. Payment may be made in
a.         any commercially acceptable means except cash.
b.   cash only.
c.         cash or any commercially acceptable substitute.
d.         cash or check only.
A10.    Metro Daily and New City Newsstand enter into a contract under which Metro agrees to deliver a certain quantity of newspapers to New City each day. The contract does not include a price term. In a suit between the par­ties over the price, a court will
a.   determine a reasonable price.
b.         impose the lowest market price.
c.         refuse to enforce the agreement.
d.         return the parties to the positions they held before the contract.
B10.    NuTech Company agrees to sell computer equipment to Office Stores, Inc. (OSI) for OSI to market to its customers. Their contract will not be en­forceable unless it includes
a.   the duration of the deal.
b.         the price of the goods.
c.         the quantity of the goods.
d.         the requirements of OSI’s customers.
A11.    Regional Products, Inc., agrees to sell to Quantity Dealers Corporation a certain amount of goods but no mention is made of where the goods are to be delivered. In general, the UCC requires that the delivery take place at
a.         a neutral place of business halfway between the parties’ locations.
b.         a UCC-designated warehouse.
c.   Regional’s place of business.
d.         Quantity’s place of business.

B11.    Doctors Medical Clinic orders 1,000 bandages from Emergency Supplies Company but fails to specify the sizes. The bandages are delivered in an assortment of sizes. Doctors Medical Clinic may
a.         accept the bandages “as is” only.
b.         accept the bandages “as is” or reject the entire shipment only.
c.   accept only the bandages that it wants and reject the rest.
d.         reject the entire shipment only.
A12.    Roy’s Chick’n Shack orders chicken from Standard Food Supplier, but Standard does not deliver. Roy’s will probably be unable to enforce the agreement if the parties
a.         did not limit the duration of the deal.
b.   did not specify a payment term.
c.         did not specify a quantity term.
d.         have not begun to perform.
B12.    Contractors Construction Corporation offers to buy from Dandy Cement Company a certain quantity of cement for a certain price. Dandy can ac­cept the offer by
a.         doing nothing.
b.         promising to ship or promptly shipping the cement.
c.   promising to ship the cement only.
d.         promptly shipping the cement only.

A13.    Desktop Company offers to sell eRetail, Inc., 1,000 computers for a $500,000, states that the offer will be open for six days, and asks for a response by fax. On the fourth day, eRetail sends an acceptance to Desktop via the mail, which is received on the sixth day. In this deal
a.   a contract is formed.
b.         no contract is formed, because Desktop asked for a response by fax.
c.         no contract is formed, because Desktop received the acceptance late.
d.         no contract is formed, because eRetail sent the acceptance late.

B13.    Best Sales, Inc., is the offeror and City Goods Corporation is the offeree under a unilateral sales contract in which Delta Products Company is also interested. Best is not notified of City’s performance within a rea­son­able time. Best
a.         may treat the offer as having lapsed.
b.         must assume that City has started to perform.
c.         must contact City.
d.   must contract with Delta.

A14.    Retail Music, Inc., offers to buy from Super Products Corporation (SPC) 1,000 blank CDs of a certain brand. Without notifying Retail, SPC timely ships CDs of a different brand. This shipment is
a.         an acceptance of the offer and a breach of the parties’ contract.
b.         an acceptance of the offer and a fulfillment of the parties’ contract.
c.         a refusal of the offer and a breach of the parties’ contract.
d.   a refusal of the offer and a fulfillment of the parties’ contract.

B14.    Equipment Rental Corporation and Family Farm, Inc., are parties to an oral agreement for a lease of goods with payments in excess of $10,000. They may satisfy the Statute of Frauds by
a.         mutually agreeing not to commit fraud.
b.         restating the terms in a phone call.
c.         setting out the terms in an e-mail.
d.   shaking hands on the deal.

A15. United Farms offers to sell Valu Bakeries, Inc., fifty bushels of wheat. Valu’s representative Wendy responds, “We agree to buy fifty bushels only if the wheat is Grade A quality.” Wendy’s statement is
a.         a breach of the parties’ contract.
b.         a counteroffer.
c.   a fulfillment of the parties’ contract.
d.         an acceptance.


B15. Kelly offers to buy cooking oil from Jim. Jim believes Kelly asks for 10,000 gallons and orally agrees to the sale. When the parties later dispute the deal in court, Jim’s claim of 10,000 gallons and Kelly’s testimony that she ordered only 1,000 gallons
     a.   prevents the enforcement of any contract between these parties.
     b.   supports an enforceable contract for 10,000 gallons.
     c.   supports an enforceable contract for 5,500 gallons.
     d.   supports an enforceable contract for 1,000 gallons.

A16.    Rite Clothiers, Inc., sells t-shirts to Brand Name Stores, Inc., under an existing con­tract. When textile costs increase, Brand agrees to a price increase, but later wants to cancel the con­tract. Brand may
a.         cancel the contract immediately.
b.         cancel the contract only after accepting a final shipment.
c.   cancel the contract only on reasonable notice.
d.         not cancel the contract.

B16.    Recreation Supplies, Inc. (RSI), and Sam, the owner of a Tourist Time shop, orally agree to a sale of beach balls and seashells for $1,000. Sam gives RSI a check for $400 as a partial payment. This contract is
a.   enforceable to the extent of $400.
b.         fully enforceable because it is for specially made goods.
c.         fully enforceable because it is oral.
d.         not enforceable.

A17.    Quinn enters into a series of agreements with Reba involving a sale of a Suite Dreams Motel, including the land, building, furnishings, shares of stock in Suite Dreams Company, and a contract with Trudy to create an ad campaign. Reba suspects that Quinn may be misrepresenting the facts. The UCC Statute of Frauds governs the sale of
a.         any of the property evidenced by a writing.
b.   any of the property that may involve fraud.
c.         the furnishings priced at $500 or more.
d.         the land and the building.

B17.    Fruits & Vegetables, Inc., and Great Grocery Stores dispute the interpretation of an ambiguous phrase in their contract. In a suit between the parties to con­strue the contract, a court may accept evidence of
a.   consistent additional terms only.
b.         consistent additional terms and contradictory terms only.
c.         contradictory terms only.
d.         anything extrinsic to the contract.

A18.    Tasty Pastries, Inc., and other bakers refer to a “baker’s dozen” as con­sisting of a collection of thirteen baked goods. This is an example of
a.   course of dealing.
b.         course of performance.
c.         square dealing.
d.         usage of trade.
B18. Timber Products, Inc., and Walt, a consumer, enter into a contract for a sale of plywood. If the contract includes a clause that is perceived as grossly unfair to Walt, its enforcement may be challenged under the doc­trine of

a.   good faith.
b.         square dealing.
c.         the mere image rule.
d.         unconscionability.
A19.    Gail enters into a contract with Hi-Price Appliances, Inc. In a suit be­tween the parties over payment under the contract, Gail claims that a certain clause is unconscionable. If the court agrees, it may
a.         enforce, limit, or refuse to enforce the contract or the disputed clause.
b.   enforce the contract without the disputed clause only.
c.         limit the application of the disputed clause only.
d.         refuse to enforce the entire contract only.


B19.    In Case 20.3, Jones v. Star Credit Corp., which of the following factors was not considered by the court when it declared a contract for the pur­chase of a freezer unconscionable
a.   The price the plaintiffs were charged was more than four times the freezer’s retail value.
b.   The credit charges alone exceeded the freezer’s retail value.
c.   The seller knew of the buyers’ limited resources.
d.   The freezer was not merchantable.

A20.    Overseas Corporation (OC), a U.S. firm, orally agrees to sell six freezers to Pisa Pizza, Ltd., in Italy. OC fails to deliver. Under the CISG, Pisa Pizza can
a.   enforce the agreement.
b.         not enforce the agreement because it is not in writing.
c.         not enforce the agreement because the price term is not specified.
d.         not enforce the agreement because there is no consideration.
B20.    Rodeo, S.A., which is based in Spain, enters into a contract for the sale of seven hydraulic lifts to Tonnage Shipping Company, which is based in the United States. This contract is governed by
a.   Spanish law.
b.         the provisions in the laws of both countries that are similar.
c.         the UCC.
d.         the United Nations Convention on Contracts for the International Sale of Goods. 

Chapter 33


Labor and Employment Law



TRUE/FALSE QUESTIONS

1.   The employment-at-will doctrine is still in widespread use.

     ANSWER:   T            PAGE:  933          TYPE:  N
MISC: AACSB Reflective      Skill Level: AICPA Legal

2.   Firing a worker who refuses to perform an illegal act violates pub­lic policy.

     ANSWER:   T            PAGE:  934          TYPE:  =
MISC: AACSB Analytic        Skill Level: AICPA Legal

3.   Whistleblower statutes protect employers from workers’ disclosure of the employer’s wrongdoing.

     ANSWER:   F            PAGE:  936          TYPE:  =
MISC: AACSB Analytic        Skill Level: AICPA Legal

4.   Whistleblower statutes allow employers to discharge employees who blow the whistle on their employers.

     ANSWER:   F            PAGE:  936          TYPE:  =
MISC: AACSB Analytic        Skill Level: AICPA Legal

5.   An employee may file an action for wrongful discharge against an em­ployer who discharges the employee in violation of an employment contract.

     ANSWER:   T            PAGE:  936          TYPE:  =
MISC: AACSB Analytic        Skill Level: AICPA Legal

6.   Children over fourteen years of age can work in hazardous jobs.

     ANSWER:   F            PAGE:  937          TYPE:  N
MISC: AACSB Analytic        Skill Level: AICPA Legal

7.   Employers may waive the overtime requirements of the Fair Labor Standards Act.

     ANSWER:   F            PAGE:  937          TYPE:  N
MISC: AACSB Analytic        Skill Level: AICPA Legal

8.   State law regulates overtime pay.

     ANSWER:   F            PAGE:  937          TYPE:  N
MISC: AACSB Analytic        Skill Level: AICPA Legal

9.   Federal labor law protects employees’ rights to picket.

     ANSWER:   T            PAGE:  939          TYPE:  +
MISC: AACSB Analytic        Skill Level: AICPA Legal

10.  Contracts limiting employees’ rights to join unions are unlawful.

     answer:   T            PAGE:  939          TYPE:  N
MISC: AACSB Analytic        Skill Level: AICPA Legal

11.  The law guarantees union members higher wages than nonunion workers.

     answer:   f            PAGE:  939          TYPE:  N
MISC: AACSB Analytic        Skill Level: AICPA Legal

12.  Employees have the right to engage in collective bargaining through elected representatives.

     ANSWER:   T            PAGE:  939          TYPE:  =
MISC: AACSB Analytic        Skill Level: AICPA Legal

13.  Federal labor laws cover all workers.

     answer:   f            PAGE:  941          TYPE:  N
MISC: AACSB Analytic        Skill Level: AICPA Legal

14.  A union shop is a workplace that requires union membership as a pre­requi­site for em­ployment.

     ANSWER:   F            PAGE:  942          TYPE:  +
MISC: AACSB Analytic        Skill Level: AICPA Legal


15.  A closed shop is a workplace that requires union membership after a worker has been on a job for specified amount of time.

     ANSWER:   F            PAGE:  942          TYPE:  +
MISC: AACSB Analytic        Skill Level: AICPA Legal

16.  During a union election campaign, an employer may not limit the activi­ties of union supporters.

     answer:   F            PAGE:  943          TYPE:  N
MISC: AACSB Analytic        Skill Level: AICPA Legal

17.  During a union election campaign, an employer may not campaign among its workers against the union.

     ANSWER:   F            PAGE:  943          TYPE:  N
MISC: AACSB Analytic        Skill Level: AICPA Legal

18.  During any strike, an employer may hire temporary substitute workers to replace strikers.

     ANSWER:   T            PAGE:  944          TYPE:  N
MISC: AACSB Analytic        Skill Level: AICPA Legal

19.  Only the states can en­force safety standards governing workplaces.

     ANSWER:   F            PAGE:  944          TYPE:  N
MISC: AACSB Analytic        Skill Level: AICPA Legal

20.  Only the states can set safety standards governing workplaces.

     ANSWER:   F            PAGE:  944          TYPE:  +
MISC: AACSB Analytic        Skill Level: AICPA Legal

21.  Employers have no general duty to keep workplaces safe.

     ANSWER:   T            PAGE:  944          TYPE:  +
MISC: AACSB Analytic        Skill Level: AICPA Legal

22.  Only the federal government can set safety standards governing workplaces.

     ANSWER:   F            PAGE:  944          TYPE:  N
MISC: AACSB Analytic        Skill Level: AICPA Legal


23.  In most states, employers who show an ability to pay claims do not need to buy workers’ compensa­tion insurance.

     ANSWER:   T            PAGE:  945          TYPE:  =
MISC: AACSB Analytic        Skill Level: AICPA Legal

24.  Recovery under state workers’ compensation law for an on-the-job injury is only possible if the injury was caused by the employer’s negligence.

     ANSWER:   F            PAGE:  946          TYPE:  =
MISC: AACSB Analytic        Skill Level: AICPA Legal

25.  The basis for contributions to help pay for an employee’s loss of income on retirement is the employee’s annual wage base.

     ANSWER:   T            PAGE:  946          TYPE:  =
MISC: AACSB Analytic        Skill Level: AICPA Legal

26.  Employers are required to establish retirement plans for their employees.

     ANSWER:   F            PAGE:  947          TYPE:  =
MISC: AACSB Analytic        Skill Level: AICPA Legal

27.  Employee contributions to pension plans vest immediately.

     ANSWER:   T            PAGE:  947          TYPE:  +
MISC: AACSB Analytic        Skill Level: AICPA Legal

28.  All unemployed workers are eligible for unemployment compensation.

     ANSWER:   F            PAGE:  947          TYPE:  =
MISC: AACSB Analytic        Skill Level: AICPA Legal

29.  A worker whose job is terminated can still participate in the em­ployer’s health plan.

     ANSWER:   T            PAGE:  948          TYPE:  =
MISC: AACSB Analytic        Skill Level: AICPA Legal

30.  Under federal law, only key employees who take temporary family or medical leave are entitled to job reinstatement.

     ANSWER:   F            PAGE:  949          TYPE:  +
MISC: AACSB Analytic        Skill Level: AICPA Legal

31.  Supervisors may be subject to personal liability for violations of the Family and Medical Leave Act of 1993.

     ANSWER:   T            PAGE:  949          TYPE:  N
MISC: AACSB Analytic        Skill Level: AICPA Legal

32.  Under federal law, employers can monitor employees’ personal commu­nications without the employees’ consent.

     ANSWER:   F            PAGE:  950          TYPE:  N
MISC: AACSB Analytic        Skill Level: AICPA Legal

33.  An employer can require an employee to take a lie-detector test when in­vestigating losses attributable to theft.

     ANSWER:   T            PAGE:  953          TYPE:  =
MISC: AACSB Analytic        Skill Level: AICPA Legal

34.  Drug testing of employees by private employers is permitted.

     ANSWER:   T            PAGE:  953          TYPE:  +
MISC: AACSB Analytic        Skill Level: AICPA Legal

35.  An employer may hire an illegal immigrant if the employer files a special form.

     ANSWER:   F            PAGE:  955          TYPE:  =
MISC: AACSB Analytic        Skill Level: AICPA Legal


MULTIPLE-CHOICE QUESTIONS

1.   Nina works as an employee for Overland Sales, Inc. To protect Nina and other employees from arbitrary discharge, courts have created excep­tions to the employment-at-will doctrine based on

a.   an implied contract theory only.
b.   a public policy theory only.
c.   implied contract and public policy theories.
d.   neither implied contract nor public policy theories.

ANSWER:   C            PAGE:  934          TYPE:  =
MISC: AACSB Reflective      Skill Level: AICPA Legal


2.   Average Manufacturing Company (AMC) employs workers, including Bob, at six locations in two states. AMC’s discharge of Bob outside the terms of an employment contract may result in

a.   AMC’s liability for damages.
b.   Bob’s deportation under the Immigration Act.
c.   discontinuance of Bob’s health-plan coverage.
d.   monitoring AMC’s communications for privacy violations.

ANSWER:   A            PAGE:  936          TYPE:  N
MISC: AACSB Reflective      Skill Level: AICPA Legal

3.   Kyle works as a forklift operator for Local Warehouse Company. Assuming that Kyle meets other requirements, the maximum number of hours that he can work per week without over­time pay is

a.   thirty-nine.
b.   forty.
c.   forty-one.
d.   unlimited.

ANSWER:   B            PAGE:  937          TYPE:  +
MISC: AACSB Reflective      Skill Level: AICPA Legal

4.   Adam, Beth, Carol, and Dan are employees of different-sized employers in different industries. A minimum wage must be paid to employees in

a.   all industries.
b.   covered industries only.
c.   no industries.
d.   small-business industries only.

ANSWER:   B            PAGE:  937          TYPE:  =
MISC: AACSB Reflective      Skill Level: AICPA Legal

5.   Ethan is seventeen years old. Under the Fair Labor Standards Act, Ethan cannot work

a.   during school hours.
b.   in a hazardous occupation.
c.   more than eighteen hours per week.
d.   without a special permit.

ANSWER:   B            PAGE:  937          TYPE:  =
MISC: AACSB Reflective      Skill Level: AICPA Legal

6.   Ilsa works as a clerk for Java Café at minimum wage. The minimum-wage rate is revised

a.   annually, according to the rate of inflation.
b.   every seven years, according to changes in the cost of living.
c.   periodically by Congress.
d.   whenever the president issues an executive order to revise the rate.

ANSWER:   C            PAGE:  937          TYPE:  N
MISC: AACSB Reflective      Skill Level: AICPA Legal

7.   To fulfill a temporary contract, Alpha Communications Company needs some employees to work overtime. All of the following employees may work more than forty hours per week except

a.   Beth, who is under sixteen and on spring break.
b.   Carl, who is an executive.
c.   Diane, who is a professional.
d.   Earl, who is an outside salesperson.

ANSWER:   A            PAGE:  937          TYPE:   N
MISC: AACSB Reflective      Skill Level: AICPA Legal

8.   Industrial Solvents, Inc. (ISI), and its employees are subject to federal la­bor law, which is concerned with the rights of

a.   ISI only.
b.   neither the employees nor ISI.
c.   the employees and ISI.
d.   the employees only.

ANSWER:   C            PAGE:  939          TYPE:  +
MISC: AACSB Reflective      Skill Level: AICPA Legal

9.   Packaged Foods Corporation (PFC) employs 1.000 workers in three loca­tions. Under the Norris-LaGuardia Act, PFC’s workers have the right to

a.   demand that PFC be a closed shop.
b.   higher pay.
c.   organize.
d.   refuse to bargain with PFC.

ANSWER:   C            PAGE:  939          TYPE:  +
MISC: AACSB Reflective      Skill Level: AICPA Legal

10.  The employees of Acme Machine, Inc., designate the Best Machinists Union (BMU) as their bargaining representative. Acme refuses to bar­gain with BMU and fires several workers for supporting the union. This violates

     a.   the Labor-Management Relations Act.
     b.   the National Labor Relations Act.
     c.   the Norris-LaGuardia Act.
     d.   no federal law.

     answer:   B            PAGE:  939          TYPE:  N
MISC: AACSB Reflective      Skill Level: AICPA Legal

11.  Cleo, who is an employee of Delta Industries, Inc., is pro­tected under the National Labor Relations Act, which is enforced by the Na­tional Labor Relations Board (NLRB). The NLRB has the authority to

a.   investigate unfair labor practices.
b.   manage state unemployment funds.
c.   organize workers into unions.
d.   oversee administration of workers’ compensation laws.

ANSWER:   A            PAGE:  940          TYPE:  +
MISC: AACSB Reflective      Skill Level: AICPA Legal

12.  The Chip Makers Union would like Digital, Inc., to require union mem­bership of its job applicants as a condition to obtaining employment. This is

a.   illegal in all states.
b.   illegal only in states that have right-to-work laws.
c.   legal in all states.
d.   legal only in states that have right-to-work laws.

ANSWER:   A            PAGE:  941          TYPE:  =
MISC: AACSB Reflective      Skill Level: AICPA Legal

13.  The Hotel Employees Union is engaged in collective bargaining with International Lodging, Inc. The employer would show bad faith by

a.   compromising on particular issues.
b.   constantly shifting positions on disputed contract terms.
c.   not discussing union positions with employees.
d.   sending only bargainers who have authority to commit to a contract.

     answer:   B            PAGE:  943          TYPE:  N
MISC: AACSB Reflective      Skill Level: AICPA Legal

14.  Sue, a clerk for a Totally Tacos restaurant, strikes with the other employ­ees. After the strike, Sue must be rehired—even if there is no work at the restaurant—if the strike was

a.   an economic strike.
b.   an economic strike or an unfair labor practice strike.
c.   an unfair labor practice strike.
d.   neither an economic strike nor an unfair labor practice strike.

ANSWER:   C            PAGE:  944          TYPE:  N
MISC: AACSB Reflective      Skill Level: AICPA Legal

15.  Quinn is an employee of Regional Industries, Inc. Quinn is threatened with a discharge when he refuses a transfer to a Regional department in several employees suffered serious injuries from exposure to hazardous chemicals. If Quinn acted in good faith, he may be entitled to protection from discharge under

a.   the Consolidated Omnibus Budget Reconciliation Act.
b.   the Family and Medical Leave Act.
c.   the Occupational Health and Safety Act.
d.   the state workers’ compensation act.

ANSWER:   C            PAGE:  944          TYPE:  =
MISC: AACSB Reflective      Skill Level: AICPA Legal

16.  Tri-State Distribution Corporation has a staff of ten, which will increase to fifty if it obtains a certain contract. Employers are required to keep oc­cu­pational injury and illness records for each employee if the employers have

a.   eleven or more employees.
b.   twenty-one or more employees.
c.   thirty-one or more employees.
d.   fifty-one or more employees.

ANSWER:   A            PAGE:  945          TYPE:  =
MISC: AACSB Reflective      Skill Level: AICPA Legal


17.  Jay works as an employee for Kitchen Appliances, Inc. (KAI), which is subject to the Occupational Safety and Health Act. Jay is killed in a work-related accident. To avoid a fine, KAI must notify

a.   the Social Security Administration.
b.   the U.S Department of Health and Human Services.
c.   the U.S. Department of Justice.
d.   the U.S. Department of Labor.

ANSWER:   D            PAGE:  945          TYPE:  N
MISC: AACSB Reflective      Skill Level: AICPA Legal

18.  Ken works for Local Excavation Corporation (LEC). While operating a backhoe, Ken suffers an injury. Ken will be compen­sated under state workers’ compensation laws only if

a.   Ken does not have medical insurance.
b.   Ken is completely disabled.
c.   Ken’s injury was accidental and occurred on the job or in the course of employment.
d.   Ken successfully sues LEC.

ANSWER:   C            PAGE:  946          TYPE:  +
MISC: AACSB Reflective      Skill Level: AICPA Legal

19.  Hugh, an employee of International Shipping Corporation (ISC), is in­jured on the job and accepts workers’ compensation. Hugh can success­fully sue ISC

a.   only if the injury was caused by ISC intentionally.
b.   only if the injury was caused by ISC’s negligence.
c.   regardless of ISC’s fault.
d.   under no circumstances.

ANSWER:   A            PAGE:  946          TYPE:  +
MISC: AACSB Reflective      Skill Level: AICPA Legal

20.  Leah is an employee of Mega Corporation. To help pay for Leah’s medical costs on retirement, contributions are required by law to be paid by

a.   Leah and Mega.
b.   Leah only.
c.   Mega only.
d.   neither Leah nor Mega.

ANSWER:   A            PAGE:  946          TYPE:  N
MISC: AACSB Reflective      Skill Level: AICPA Legal

21.  Alex is an employee of Bigg Corporation. To help pay for employees’ loss of income on retirement, contributions are required by law to be paid by

a.   Alex and Bigg.
b.   Alex only.
c.   Bigg only.
d.   neither Alex nor Bigg.

ANSWER:   A            PAGE:  946          TYPE:  N
MISC: AACSB Reflective      Skill Level: AICPA Legal

22.  Lila retires from her job as a systems analyst for Marketing Solutions, Inc. Lila’s Social Security benefits are fixed by statute and will

a.   automatically increase with the cost of living.
b.   increase with the cost of living only when the statute is “re-fixed.”
c.   increase with the cost of living whenever revenues increase.
d.   not increase with the cost of living.

ANSWER:   A            PAGE:  946          TYPE:  N
MISC: AACSB Reflective      Skill Level: AICPA Legal

23.  Luke is a maintenance employee for Natural Products Company. Under the Employee Retirement Income Security Act, Luke’s contributions to his pension plan vest

a.   after five years of employment.
b.   after five years of retirement.
c.   five years before retirement.
d.   immediately.

ANSWER:   D            PAGE:  947          TYPE:  N
MISC: AACSB Reflective      Skill Level: AICPA Legal

24.  Mia works as an employee for National Interstate Industries, Inc. (NIII). Owen, who is unemployed, collects unemployment compensa­tion. This is provided by a tax on

a.   Mia and Owen only.
b.   Mia, NIII, and Owen.
c.   NIII only.
d.   not Mia, NIII or Owen.

ANSWER:   C            PAGE:  947          TYPE:  +
MISC: AACSB Reflective      Skill Level: AICPA Legal

25.  Machine Manufacturing, Inc., employs four hundred workers at three locations in three states. Excluded from protection under the Consoli­dated Omnibus Budget Reconciliation Act (COBRA) of 1985 are workers who

a.   are fired for gross misconduct.
b.   are laid off for budgetary reasons.
c.   have their hours decreased from full-time to part-time.
d.   voluntarily quit their jobs.

ANSWER:   A            PAGE:  948          TYPE:  =
MISC: AACSB Reflective      Skill Level: AICPA Legal

26.  Consolidated Computers, Inc., lays off fifty of its five hundred workers, including Don. Consolidated can eliminate Don’s medical insurance coverage

a.   if Consolidated changes its group insurer.
b.   if Consolidated completely eliminates its group benefit plan.
c.   if Don files a suit against Consolidated for wrongful discharge.
d.   under any circumstances.

ANSWER:   B            PAGE:  948          TYPE:  =
MISC: AACSB Reflective      Skill Level: AICPA Legal

27.  Café Dining, Inc., employs one hundred workers in at three loca­tions in two states. Under the Family and Medical Leave Act of 1993, Café must provide its employees, during any twelve-month period, family or medi­cal leave of up to

a.   twelve days.
b.   twelve weeks.
c.   twelve months.
d.   twelve years.

ANSWER:   B            PAGE:  949          TYPE:  =
MISC: AACSB Reflective      Skill Level: AICPA Legal


28.  Holly takes temporary leave from her job at Interstate Assembly Company to care for her new baby. When she attempts to return to work, Interstate refuses to reinstate her. Under the Family and Medical Leave Act, Holly may be entitled to

a.   damages only.
b.   damages or job reinstatement only.
c.   double damages, job reinstatement, a promotion, and more.
d.   none of the above.

ANSWER:   C            PAGE:  949          TYPE:  =
MISC: AACSB Reflective      Skill Level: AICPA Legal

29.  Allied Industries, Inc., wants to monitor its employees’ electronic com­munications. To avoid liability under laws related to employee monitor­ing, Allied should discuss the monitoring with

a.   no one.
b.   the employees.
c.   the government.
d.   the public generally.

ANSWER:   B            PAGE:  951          TYPE:  N
MISC: AACSB Reflective      Skill Level: AICPA Legal

30.  First National Bank may subject its employees to lie-detector tests when investigating

a.   health and medical conditions.
b.   losses attributable to theft.
c.   prior work history.
d.   suspected drug use.

ANSWER:   B            PAGE:  952          TYPE:  =
MISC: AACSB Reflective      Skill Level: AICPA Legal

31.  Security Guards, Inc. (SGI), is a security service firm. SGI may subject its employees to

a.   drug tests and lie-detector tests.
b.   drug tests only.
c.   lie-detector tests only.
d.   neither drug tests nor lie-detector tests.

ANSWER:   A            PAGE:  952          TYPE:  N
MISC: AACSB Reflective      Skill Level: AICPA Legal

32.  Leo works as an employee for Maxim Corporation, a private em­ployer. Maxim announces that it will start random drug testing of its employees. To resist this policy, Leo may look for protection under

a.   a federal administrative agency rule.
b.   a state constitution or statute.
c.   the U.S. Constitution.
d.   none of the above.

ANSWER:   B            PAGE:  953          TYPE:  +
MISC: AACSB Reflective      Skill Level: AICPA Legal

33.  A+ Accounting Corporation, a private employer, handles bookkeeping for small employers. In most circumstances, with exceptions, federal law clearly pro­hibits Omega from subjecting its employees to

a.   AIDS tests.
b.   electronic monitoring of business communications.
c.   genetic tests.
d.   lie-detector tests.

ANSWER:   C            PAGE:  955          TYPE:  N
MISC: AACSB Reflective      Skill Level: AICPA Legal

34.  Fruits & Vegetables, Inc., employs hundreds of seasonal and permanent workers, both skilled and unskilled, in seven states. Under the Immigration Act of 1990, Fruits & Vegetables can hire illegal immigrants

a.   if either the employer or the immigrants file special forms.
b.   only if the employer files a special form.
c.   only if the immigrants file special forms.
d.   under no circumstances.

ANSWER:   D            PAGE:  955          TYPE:  +
MISC: AACSB Reflective      Skill Level: AICPA Legal

35.  Silicon Software, Inc., recruits employees in India and other Asian coun­tries. The Immigration Act of 1990

a.   encourages Silicon to hire illegal immigrants.
b.   encourages skilled workers to enter the United States.
c.   requires Silicon to hire illegal immigrants.
d.   requires skilled workers to enter the United States.

ANSWER:   B            PAGE:  955          TYPE:  =
MISC: AACSB Reflective      Skill Level: AICPA Legal


Essay Questions

1.   Collective bargaining between Vital Company and Workers Union (WU) reaches an impasse. WU calls a strike against Vital to pressure it into making concessions. The unionized workers leave their jobs and refuse to work. The workers picket Vital’s plant, standing outside the facility with signs that complain of its management’s unfairness. Is this action legal? If so, do the workers have the right to be paid while they are striking? Can non-workers participate in the picketing? Can co-workers refuse to cross the picket line?

     ANSWER:     The strike described in this question is legal. A strike is an extreme action, and striking workers lose the right to be paid. A strike may be based on a union’s desire for a better contract or an employer’s engaging in an unfair labor practice. Federal labor law regulates the conduct of strike, but guarantees the right to strike, including picketing, which is protected by the free speech guarantee of the First Amendment to the U.S. Constitution. Non-workers can participate in the picketing of an employer. The National Labor Relations Act also gives workers the right to refuse to cross a picket line of co-workers who are involved in a lawful strike.

PAGE:     944                               type:  N
  MISC: AACSB Reflective         Skill Level: AICPA Decision
                                 Modeling

2.   Elin works for Formative Molding Company. Near her workstation is a conveyor belt that runs through a large industrial oven. Some workers, including Greg, Elin’s supervisor, use the oven to heat food. Thirty-inch-high flasks containing molds are fixed at regular intervals on the con­veyor and transported into the oven. Elin walks between the flasks to get to her workstation. One day, when the conveyor is not moving, Elin uses the oven to cook a frozen pot pie. As she removes the pie from the oven, the conveyor comes on. One of the flasks strikes Elin, seriously injuring her. Elin seeks recovery under the state workers’ compensation law. Should she recover? Why or why not?

     ANSWER:     For a worker to be eligible for workers’ compensation, the injury must have been an accident and have arisen out of or in the course of employment. Although Elin was accidentally struck by the flask, re­cov­ery could be denied because Elin was not engaged in an activ­ity di­rectly related to her work but was instead heating frozen food in an indus­trial oven. By the same token, how­ever, it is a common practice to use the oven and the employer has apparently not forbidden the practice. Be­cause the cooking is done by many em­ployees, including Elin’s super­visor, Formative can be imputed with the knowledge of it. On balance, then, Elin should be al­lowed to receive workers’ compensation benefits.

PAGES:    945–946                           type:  N
  MISC: AACSB Reflective         Skill Level: AICPA Decision
                                  Modeling

Không có nhận xét nào:

Đăng nhận xét